Combinatorial proof of summation of $sumlimits_{k = 0}^n {n choose k}^2= {2n choose n}$












57












$begingroup$


I was hoping to find a more "mathematical" proof, instead of proving logically $displaystyle sum_{k = 0}^n {n choose k}^2= {2n choose n}$.



I already know the logical Proof:



$${n choose k}^2 = {n choose k}{ n choose n-k}$$



Hence summation can be expressed as:



$$binom{n}{0}binom{n}{n} + binom{n}{1}binom{n}{n-1} + cdots + binom{n}{n}binom{n}{0}$$



One can think of it as choosing $n$ people from a group of $2n$
(imagine dividing a group of $2n$ into $2$ groups of $n$ people each. I can get $k$ people from group $1$ and another $n-k$ people from group $2$. We do this from $k = 0$ to $n$.










share|cite|improve this question











$endgroup$








  • 16




    $begingroup$
    Just FYI, what you call a "logical proof" is known as a "combinatorial proof", and such a proof is perfectly valid and often very insightful. What I suspect you mean by "mathematical proof" is one dealing with the numerical structure of sums and combinations, which would be better called an "analytical proof". Both styles of proof are equally mathematical.
    $endgroup$
    – Austin Mohr
    May 23 '12 at 4:57








  • 6




    $begingroup$
    This is secretly subsumed by this question
    $endgroup$
    – davidlowryduda
    May 23 '12 at 5:00








  • 1




    $begingroup$
    You could obtain the same combinatorial proof by noting that $binom{2n}{n}$ counts the number of paths from $(0,0)$ to $(n,n)$ on an $ntimes n$ grid.
    $endgroup$
    – Holdsworth88
    May 23 '12 at 6:03






  • 7




    $begingroup$
    I think your combinatorial proof is really nice, and you should not be unhappy with it.
    $endgroup$
    – MJD
    May 23 '12 at 13:54








  • 2




    $begingroup$
    Incidentally, this is a special case of math.stackexchange.com/questions/337923/….
    $endgroup$
    – Greek - Area 51 Proposal
    Nov 16 '13 at 10:26
















57












$begingroup$


I was hoping to find a more "mathematical" proof, instead of proving logically $displaystyle sum_{k = 0}^n {n choose k}^2= {2n choose n}$.



I already know the logical Proof:



$${n choose k}^2 = {n choose k}{ n choose n-k}$$



Hence summation can be expressed as:



$$binom{n}{0}binom{n}{n} + binom{n}{1}binom{n}{n-1} + cdots + binom{n}{n}binom{n}{0}$$



One can think of it as choosing $n$ people from a group of $2n$
(imagine dividing a group of $2n$ into $2$ groups of $n$ people each. I can get $k$ people from group $1$ and another $n-k$ people from group $2$. We do this from $k = 0$ to $n$.










share|cite|improve this question











$endgroup$








  • 16




    $begingroup$
    Just FYI, what you call a "logical proof" is known as a "combinatorial proof", and such a proof is perfectly valid and often very insightful. What I suspect you mean by "mathematical proof" is one dealing with the numerical structure of sums and combinations, which would be better called an "analytical proof". Both styles of proof are equally mathematical.
    $endgroup$
    – Austin Mohr
    May 23 '12 at 4:57








  • 6




    $begingroup$
    This is secretly subsumed by this question
    $endgroup$
    – davidlowryduda
    May 23 '12 at 5:00








  • 1




    $begingroup$
    You could obtain the same combinatorial proof by noting that $binom{2n}{n}$ counts the number of paths from $(0,0)$ to $(n,n)$ on an $ntimes n$ grid.
    $endgroup$
    – Holdsworth88
    May 23 '12 at 6:03






  • 7




    $begingroup$
    I think your combinatorial proof is really nice, and you should not be unhappy with it.
    $endgroup$
    – MJD
    May 23 '12 at 13:54








  • 2




    $begingroup$
    Incidentally, this is a special case of math.stackexchange.com/questions/337923/….
    $endgroup$
    – Greek - Area 51 Proposal
    Nov 16 '13 at 10:26














57












57








57


37



$begingroup$


I was hoping to find a more "mathematical" proof, instead of proving logically $displaystyle sum_{k = 0}^n {n choose k}^2= {2n choose n}$.



I already know the logical Proof:



$${n choose k}^2 = {n choose k}{ n choose n-k}$$



Hence summation can be expressed as:



$$binom{n}{0}binom{n}{n} + binom{n}{1}binom{n}{n-1} + cdots + binom{n}{n}binom{n}{0}$$



One can think of it as choosing $n$ people from a group of $2n$
(imagine dividing a group of $2n$ into $2$ groups of $n$ people each. I can get $k$ people from group $1$ and another $n-k$ people from group $2$. We do this from $k = 0$ to $n$.










share|cite|improve this question











$endgroup$




I was hoping to find a more "mathematical" proof, instead of proving logically $displaystyle sum_{k = 0}^n {n choose k}^2= {2n choose n}$.



I already know the logical Proof:



$${n choose k}^2 = {n choose k}{ n choose n-k}$$



Hence summation can be expressed as:



$$binom{n}{0}binom{n}{n} + binom{n}{1}binom{n}{n-1} + cdots + binom{n}{n}binom{n}{0}$$



One can think of it as choosing $n$ people from a group of $2n$
(imagine dividing a group of $2n$ into $2$ groups of $n$ people each. I can get $k$ people from group $1$ and another $n-k$ people from group $2$. We do this from $k = 0$ to $n$.







combinatorics summation binomial-coefficients combinatorial-proofs






share|cite|improve this question















share|cite|improve this question













share|cite|improve this question




share|cite|improve this question








edited May 9 '18 at 4:59









Greek - Area 51 Proposal

3,168769103




3,168769103










asked May 23 '12 at 4:52









Lance CLance C

286143




286143








  • 16




    $begingroup$
    Just FYI, what you call a "logical proof" is known as a "combinatorial proof", and such a proof is perfectly valid and often very insightful. What I suspect you mean by "mathematical proof" is one dealing with the numerical structure of sums and combinations, which would be better called an "analytical proof". Both styles of proof are equally mathematical.
    $endgroup$
    – Austin Mohr
    May 23 '12 at 4:57








  • 6




    $begingroup$
    This is secretly subsumed by this question
    $endgroup$
    – davidlowryduda
    May 23 '12 at 5:00








  • 1




    $begingroup$
    You could obtain the same combinatorial proof by noting that $binom{2n}{n}$ counts the number of paths from $(0,0)$ to $(n,n)$ on an $ntimes n$ grid.
    $endgroup$
    – Holdsworth88
    May 23 '12 at 6:03






  • 7




    $begingroup$
    I think your combinatorial proof is really nice, and you should not be unhappy with it.
    $endgroup$
    – MJD
    May 23 '12 at 13:54








  • 2




    $begingroup$
    Incidentally, this is a special case of math.stackexchange.com/questions/337923/….
    $endgroup$
    – Greek - Area 51 Proposal
    Nov 16 '13 at 10:26














  • 16




    $begingroup$
    Just FYI, what you call a "logical proof" is known as a "combinatorial proof", and such a proof is perfectly valid and often very insightful. What I suspect you mean by "mathematical proof" is one dealing with the numerical structure of sums and combinations, which would be better called an "analytical proof". Both styles of proof are equally mathematical.
    $endgroup$
    – Austin Mohr
    May 23 '12 at 4:57








  • 6




    $begingroup$
    This is secretly subsumed by this question
    $endgroup$
    – davidlowryduda
    May 23 '12 at 5:00








  • 1




    $begingroup$
    You could obtain the same combinatorial proof by noting that $binom{2n}{n}$ counts the number of paths from $(0,0)$ to $(n,n)$ on an $ntimes n$ grid.
    $endgroup$
    – Holdsworth88
    May 23 '12 at 6:03






  • 7




    $begingroup$
    I think your combinatorial proof is really nice, and you should not be unhappy with it.
    $endgroup$
    – MJD
    May 23 '12 at 13:54








  • 2




    $begingroup$
    Incidentally, this is a special case of math.stackexchange.com/questions/337923/….
    $endgroup$
    – Greek - Area 51 Proposal
    Nov 16 '13 at 10:26








16




16




$begingroup$
Just FYI, what you call a "logical proof" is known as a "combinatorial proof", and such a proof is perfectly valid and often very insightful. What I suspect you mean by "mathematical proof" is one dealing with the numerical structure of sums and combinations, which would be better called an "analytical proof". Both styles of proof are equally mathematical.
$endgroup$
– Austin Mohr
May 23 '12 at 4:57






$begingroup$
Just FYI, what you call a "logical proof" is known as a "combinatorial proof", and such a proof is perfectly valid and often very insightful. What I suspect you mean by "mathematical proof" is one dealing with the numerical structure of sums and combinations, which would be better called an "analytical proof". Both styles of proof are equally mathematical.
$endgroup$
– Austin Mohr
May 23 '12 at 4:57






6




6




$begingroup$
This is secretly subsumed by this question
$endgroup$
– davidlowryduda
May 23 '12 at 5:00






$begingroup$
This is secretly subsumed by this question
$endgroup$
– davidlowryduda
May 23 '12 at 5:00






1




1




$begingroup$
You could obtain the same combinatorial proof by noting that $binom{2n}{n}$ counts the number of paths from $(0,0)$ to $(n,n)$ on an $ntimes n$ grid.
$endgroup$
– Holdsworth88
May 23 '12 at 6:03




$begingroup$
You could obtain the same combinatorial proof by noting that $binom{2n}{n}$ counts the number of paths from $(0,0)$ to $(n,n)$ on an $ntimes n$ grid.
$endgroup$
– Holdsworth88
May 23 '12 at 6:03




7




7




$begingroup$
I think your combinatorial proof is really nice, and you should not be unhappy with it.
$endgroup$
– MJD
May 23 '12 at 13:54






$begingroup$
I think your combinatorial proof is really nice, and you should not be unhappy with it.
$endgroup$
– MJD
May 23 '12 at 13:54






2




2




$begingroup$
Incidentally, this is a special case of math.stackexchange.com/questions/337923/….
$endgroup$
– Greek - Area 51 Proposal
Nov 16 '13 at 10:26




$begingroup$
Incidentally, this is a special case of math.stackexchange.com/questions/337923/….
$endgroup$
– Greek - Area 51 Proposal
Nov 16 '13 at 10:26










6 Answers
6






active

oldest

votes


















25












$begingroup$

The combinatorial explanation is straightforward. There's also a roundabout approach through what are called "generating functions." The binomial theorem tells us that



$$(1+x)^n(x+1)^n=left(sum_{a=0}^nbinom{n}{a}x^aright)left(sum_{b=0}^nbinom{n}{b}x^{n-b}right)=sum_{c=0}^{2n}left(sum_{a+n-b=c}binom{n}{a}binom{n}{b}right)x^c$$



The $x^n$ coefficient of the above occurs with $c=n$, wherein the coefficient is



$$sum_{a+n-b=n}binom{n}{a}binom{n}{b}=sum_{a=0}^nbinom{n}{a}^2.$$



However, the $x^n$ coefficient of $(1+x)^n(x+1)^n=(1+x)^{2n}$, again by the binomial theorem, is



$$binom{2n}{n}. $$



Equating the two gives the result.






share|cite|improve this answer









$endgroup$













  • $begingroup$
    +1. @Lance C Do you see that this is the same as your "logical" proof? Choosing $n$ people from $2n$ is what you are doing when you look at the coefficient of $x^n$ in the expansion.
    $endgroup$
    – user17762
    May 23 '12 at 5:12










  • $begingroup$
    Can you explain me in more detail... why $sum_{a=0}^nbinom{n}{a}x^a $ * $sum_{b=0}^nbinom{n}{b}x^{n-b}$ is equal to $sum_{c=0}^{2n} sum_{a+n-b=c}binom{n}{a}binom{n}{b} x^c$ please
    $endgroup$
    – Salvattore
    Jul 25 '14 at 4:59












  • $begingroup$
    @Salvattore $sum_{a,b}binom{n}{a}binom{n}{b}x^{a+n-b}$, then collect all the coefficients of $x^c$ for fixed $c$ into $sum_{a+n-b=c}binom{n}{a}binom{n}{b}$ to get $sum_cleft[sum_{a+n-b=c}binom{n}{a}binom{n}{b}right]x^c$. If you want to work with generating functions, or even just plain polynomials, you need to be able to collect like terms, it is virtually a requirement.
    $endgroup$
    – anon
    Jul 29 '14 at 17:23





















19












$begingroup$

Since $dbinom n k= dbinom n {n-k}$, the identity
$$
sum_{k=0}^n binom n k ^2 = binom {2n} n
$$
is the same as
$$
sum_{k=0}^n binom n k binom n {n-k} = binom {2n} n.
$$
So say a committee consists of $n$ Democrats and $n$ Republicans, and one will choose a subcommittee of $n$ members. One may choose $k$ Democrats and $n-k$ Republicans in $dbinom n k cdot dbinom n {n-k}$ ways. The number of Democrats is in the set ${0,1,2,ldots,n}$, thus ranging from all Republicans to all Democrats. The sum then gives the total number of ways to choose $n$ out of $2n$.






share|cite|improve this answer









$endgroup$





















    12












    $begingroup$

    Consider the graph underlying Pascal's triangle:
    Pascal's triangle



    In this graph, the number at each node is a binomial coefficient and can also be thought of as the number of downward paths from the apex to that node.



    The left side of the identity is the number of paths that start at the apex, go down to the $n$th row and return to the apex (let's call them round trips to $n$). By reflecting the return path about the $n$th row, we get a bijective correspondence between return trips to $n$ and paths from the apex to the central node of the $2n$th row. This is nothing but the central binomial coefficient $binom{2n}n$.






    share|cite|improve this answer









    $endgroup$





















      11












      $begingroup$

      $newcommand{angles}[1]{leftlangle, #1 ,rightrangle}
      newcommand{braces}[1]{leftlbrace, #1 ,rightrbrace}
      newcommand{bracks}[1]{leftlbrack, #1 ,rightrbrack}
      newcommand{ceil}[1]{,leftlceil, #1 ,rightrceil,}
      newcommand{dd}{{rm d}}
      newcommand{ds}[1]{displaystyle{#1}}
      newcommand{expo}[1]{,{rm e}^{#1},}
      newcommand{fermi}{,{rm f}}
      newcommand{floor}[1]{,leftlfloor #1 rightrfloor,}
      newcommand{half}{{1 over 2}}
      newcommand{ic}{{rm i}}
      newcommand{iff}{Longleftrightarrow}
      newcommand{imp}{Longrightarrow}
      newcommand{pars}[1]{left(, #1 ,right)}
      newcommand{partiald}[3]{frac{partial^{#1} #2}{partial #3^{#1}}}
      newcommand{pp}{{cal P}}
      newcommand{root}[2]{,sqrt[#1]{vphantom{large A},#2,},}
      newcommand{sech}{,{rm sech}}
      newcommand{sgn}{,{rm sgn}}
      newcommand{totald}[3]{frac{{rm d}^{#1} #2}{{rm d} #3^{#1}}}
      newcommand{ul}[1]{underline{#1}}
      newcommand{verts}[1]{leftvert, #1 ,rightvert}$

      begin{align}
      color{#66f}{largesum_{k = 0}^{n}{n choose k}^{2}}&=
      sum_{k = 0}^{n}{n choose k}
      oint_{verts{z} = 1}{pars{1 + z}^{n} over z^{k + 1}},{dd z over 2piic}
      \[5mm] & =
      oint_{verts{z} = 1}{pars{1 + z}^{n} over z}
      sum_{k = 0}^{n}{n choose k}pars{1 over z}^{k},{dd z over 2piic}
      \[5mm]&=oint_{verts{z} = 1}{pars{1 + z}^{n} over z}
      pars{1 + {1 over z}}^{n},{dd z over 2piic}
      \[5mm] & =
      oint_{verts{z} = 1}{pars{1 + z}^{2n} over z^{n + 1}},{dd z over 2piic}
      \[5mm] & =color{#66f}{large{2n choose n}}
      end{align}






      share|cite|improve this answer











      $endgroup$













      • $begingroup$
        Can we change integral and summation? What is the reason behind that ?
        $endgroup$
        – Siddhant Trivedi
        Jan 4 '17 at 4:23










      • $begingroup$
        @SiddhantTrivedi In this case, it's just a $finite$ sum.
        $endgroup$
        – Felix Marin
        Jan 5 '17 at 22:27



















      1












      $begingroup$



      Okey this one is easy(if you seen this one once)
      We have to show:
      $${n choose 0}^2+{n choose 1}^2+cdots+{n choose n}^2={2n choose n} $$
      Define a set A, $$A={a_1,ldots,a_n,a_{n+1},ldots,a_{2n}}$$ consisting of $2n$ - Elements. Now we declare $V$, is a set of $n$-sets of $A$. Obviously the cardinality of $V,$ $$|V|={2n choose n}.$$ Since one can choose $n$ Elements from $2n$ Elements in $${2n choose n}$$ ways.



      Okey now we have the right side.
      The for the left side you have give a disjunctive partition of the $V$ set. This should be done in the following way:
      $V_i$ has exactly $i$-Elements from ${a_1,ldots,a_n}$, then the cardinality
      $|V_i|={n choose i}{n choose n-i}={n choose i}{n choose i}={n choose i}^2$. Then with the Rule of Sum we have the left side. And we are done.
      The Tricky Part ist to create the Partition to use the rule of sum.






      share|cite|improve this answer











      $endgroup$





















        0












        $begingroup$

        Imagine that we are distributing $n$ indistinguishable candies to $k$ distinguishable children. By a direct application of Balls and Urns, there are $${n+k-1choose k-1}$$ ways to do this. Alternatively, we can first give $0le ile n$ candies to the oldest child so that we are essentially giving $n-i$ candies to $k-1$ kids and again, with Balls and Urns, $${n+k-1choose k-1}=sum_{i=0}^n{n+k-2-ichoose k-2}$$, which simplifies to the desired result.






        share|cite|improve this answer









        $endgroup$













          Your Answer





          StackExchange.ifUsing("editor", function () {
          return StackExchange.using("mathjaxEditing", function () {
          StackExchange.MarkdownEditor.creationCallbacks.add(function (editor, postfix) {
          StackExchange.mathjaxEditing.prepareWmdForMathJax(editor, postfix, [["$", "$"], ["\\(","\\)"]]);
          });
          });
          }, "mathjax-editing");

          StackExchange.ready(function() {
          var channelOptions = {
          tags: "".split(" "),
          id: "69"
          };
          initTagRenderer("".split(" "), "".split(" "), channelOptions);

          StackExchange.using("externalEditor", function() {
          // Have to fire editor after snippets, if snippets enabled
          if (StackExchange.settings.snippets.snippetsEnabled) {
          StackExchange.using("snippets", function() {
          createEditor();
          });
          }
          else {
          createEditor();
          }
          });

          function createEditor() {
          StackExchange.prepareEditor({
          heartbeatType: 'answer',
          autoActivateHeartbeat: false,
          convertImagesToLinks: true,
          noModals: true,
          showLowRepImageUploadWarning: true,
          reputationToPostImages: 10,
          bindNavPrevention: true,
          postfix: "",
          imageUploader: {
          brandingHtml: "Powered by u003ca class="icon-imgur-white" href="https://imgur.com/"u003eu003c/au003e",
          contentPolicyHtml: "User contributions licensed under u003ca href="https://creativecommons.org/licenses/by-sa/3.0/"u003ecc by-sa 3.0 with attribution requiredu003c/au003e u003ca href="https://stackoverflow.com/legal/content-policy"u003e(content policy)u003c/au003e",
          allowUrls: true
          },
          noCode: true, onDemand: true,
          discardSelector: ".discard-answer"
          ,immediatelyShowMarkdownHelp:true
          });


          }
          });














          draft saved

          draft discarded


















          StackExchange.ready(
          function () {
          StackExchange.openid.initPostLogin('.new-post-login', 'https%3a%2f%2fmath.stackexchange.com%2fquestions%2f148583%2fcombinatorial-proof-of-summation-of-sum-limits-k-0n-n-choose-k2-2n%23new-answer', 'question_page');
          }
          );

          Post as a guest















          Required, but never shown

























          6 Answers
          6






          active

          oldest

          votes








          6 Answers
          6






          active

          oldest

          votes









          active

          oldest

          votes






          active

          oldest

          votes









          25












          $begingroup$

          The combinatorial explanation is straightforward. There's also a roundabout approach through what are called "generating functions." The binomial theorem tells us that



          $$(1+x)^n(x+1)^n=left(sum_{a=0}^nbinom{n}{a}x^aright)left(sum_{b=0}^nbinom{n}{b}x^{n-b}right)=sum_{c=0}^{2n}left(sum_{a+n-b=c}binom{n}{a}binom{n}{b}right)x^c$$



          The $x^n$ coefficient of the above occurs with $c=n$, wherein the coefficient is



          $$sum_{a+n-b=n}binom{n}{a}binom{n}{b}=sum_{a=0}^nbinom{n}{a}^2.$$



          However, the $x^n$ coefficient of $(1+x)^n(x+1)^n=(1+x)^{2n}$, again by the binomial theorem, is



          $$binom{2n}{n}. $$



          Equating the two gives the result.






          share|cite|improve this answer









          $endgroup$













          • $begingroup$
            +1. @Lance C Do you see that this is the same as your "logical" proof? Choosing $n$ people from $2n$ is what you are doing when you look at the coefficient of $x^n$ in the expansion.
            $endgroup$
            – user17762
            May 23 '12 at 5:12










          • $begingroup$
            Can you explain me in more detail... why $sum_{a=0}^nbinom{n}{a}x^a $ * $sum_{b=0}^nbinom{n}{b}x^{n-b}$ is equal to $sum_{c=0}^{2n} sum_{a+n-b=c}binom{n}{a}binom{n}{b} x^c$ please
            $endgroup$
            – Salvattore
            Jul 25 '14 at 4:59












          • $begingroup$
            @Salvattore $sum_{a,b}binom{n}{a}binom{n}{b}x^{a+n-b}$, then collect all the coefficients of $x^c$ for fixed $c$ into $sum_{a+n-b=c}binom{n}{a}binom{n}{b}$ to get $sum_cleft[sum_{a+n-b=c}binom{n}{a}binom{n}{b}right]x^c$. If you want to work with generating functions, or even just plain polynomials, you need to be able to collect like terms, it is virtually a requirement.
            $endgroup$
            – anon
            Jul 29 '14 at 17:23


















          25












          $begingroup$

          The combinatorial explanation is straightforward. There's also a roundabout approach through what are called "generating functions." The binomial theorem tells us that



          $$(1+x)^n(x+1)^n=left(sum_{a=0}^nbinom{n}{a}x^aright)left(sum_{b=0}^nbinom{n}{b}x^{n-b}right)=sum_{c=0}^{2n}left(sum_{a+n-b=c}binom{n}{a}binom{n}{b}right)x^c$$



          The $x^n$ coefficient of the above occurs with $c=n$, wherein the coefficient is



          $$sum_{a+n-b=n}binom{n}{a}binom{n}{b}=sum_{a=0}^nbinom{n}{a}^2.$$



          However, the $x^n$ coefficient of $(1+x)^n(x+1)^n=(1+x)^{2n}$, again by the binomial theorem, is



          $$binom{2n}{n}. $$



          Equating the two gives the result.






          share|cite|improve this answer









          $endgroup$













          • $begingroup$
            +1. @Lance C Do you see that this is the same as your "logical" proof? Choosing $n$ people from $2n$ is what you are doing when you look at the coefficient of $x^n$ in the expansion.
            $endgroup$
            – user17762
            May 23 '12 at 5:12










          • $begingroup$
            Can you explain me in more detail... why $sum_{a=0}^nbinom{n}{a}x^a $ * $sum_{b=0}^nbinom{n}{b}x^{n-b}$ is equal to $sum_{c=0}^{2n} sum_{a+n-b=c}binom{n}{a}binom{n}{b} x^c$ please
            $endgroup$
            – Salvattore
            Jul 25 '14 at 4:59












          • $begingroup$
            @Salvattore $sum_{a,b}binom{n}{a}binom{n}{b}x^{a+n-b}$, then collect all the coefficients of $x^c$ for fixed $c$ into $sum_{a+n-b=c}binom{n}{a}binom{n}{b}$ to get $sum_cleft[sum_{a+n-b=c}binom{n}{a}binom{n}{b}right]x^c$. If you want to work with generating functions, or even just plain polynomials, you need to be able to collect like terms, it is virtually a requirement.
            $endgroup$
            – anon
            Jul 29 '14 at 17:23
















          25












          25








          25





          $begingroup$

          The combinatorial explanation is straightforward. There's also a roundabout approach through what are called "generating functions." The binomial theorem tells us that



          $$(1+x)^n(x+1)^n=left(sum_{a=0}^nbinom{n}{a}x^aright)left(sum_{b=0}^nbinom{n}{b}x^{n-b}right)=sum_{c=0}^{2n}left(sum_{a+n-b=c}binom{n}{a}binom{n}{b}right)x^c$$



          The $x^n$ coefficient of the above occurs with $c=n$, wherein the coefficient is



          $$sum_{a+n-b=n}binom{n}{a}binom{n}{b}=sum_{a=0}^nbinom{n}{a}^2.$$



          However, the $x^n$ coefficient of $(1+x)^n(x+1)^n=(1+x)^{2n}$, again by the binomial theorem, is



          $$binom{2n}{n}. $$



          Equating the two gives the result.






          share|cite|improve this answer









          $endgroup$



          The combinatorial explanation is straightforward. There's also a roundabout approach through what are called "generating functions." The binomial theorem tells us that



          $$(1+x)^n(x+1)^n=left(sum_{a=0}^nbinom{n}{a}x^aright)left(sum_{b=0}^nbinom{n}{b}x^{n-b}right)=sum_{c=0}^{2n}left(sum_{a+n-b=c}binom{n}{a}binom{n}{b}right)x^c$$



          The $x^n$ coefficient of the above occurs with $c=n$, wherein the coefficient is



          $$sum_{a+n-b=n}binom{n}{a}binom{n}{b}=sum_{a=0}^nbinom{n}{a}^2.$$



          However, the $x^n$ coefficient of $(1+x)^n(x+1)^n=(1+x)^{2n}$, again by the binomial theorem, is



          $$binom{2n}{n}. $$



          Equating the two gives the result.







          share|cite|improve this answer












          share|cite|improve this answer



          share|cite|improve this answer










          answered May 23 '12 at 5:07









          anonanon

          72k5110213




          72k5110213












          • $begingroup$
            +1. @Lance C Do you see that this is the same as your "logical" proof? Choosing $n$ people from $2n$ is what you are doing when you look at the coefficient of $x^n$ in the expansion.
            $endgroup$
            – user17762
            May 23 '12 at 5:12










          • $begingroup$
            Can you explain me in more detail... why $sum_{a=0}^nbinom{n}{a}x^a $ * $sum_{b=0}^nbinom{n}{b}x^{n-b}$ is equal to $sum_{c=0}^{2n} sum_{a+n-b=c}binom{n}{a}binom{n}{b} x^c$ please
            $endgroup$
            – Salvattore
            Jul 25 '14 at 4:59












          • $begingroup$
            @Salvattore $sum_{a,b}binom{n}{a}binom{n}{b}x^{a+n-b}$, then collect all the coefficients of $x^c$ for fixed $c$ into $sum_{a+n-b=c}binom{n}{a}binom{n}{b}$ to get $sum_cleft[sum_{a+n-b=c}binom{n}{a}binom{n}{b}right]x^c$. If you want to work with generating functions, or even just plain polynomials, you need to be able to collect like terms, it is virtually a requirement.
            $endgroup$
            – anon
            Jul 29 '14 at 17:23




















          • $begingroup$
            +1. @Lance C Do you see that this is the same as your "logical" proof? Choosing $n$ people from $2n$ is what you are doing when you look at the coefficient of $x^n$ in the expansion.
            $endgroup$
            – user17762
            May 23 '12 at 5:12










          • $begingroup$
            Can you explain me in more detail... why $sum_{a=0}^nbinom{n}{a}x^a $ * $sum_{b=0}^nbinom{n}{b}x^{n-b}$ is equal to $sum_{c=0}^{2n} sum_{a+n-b=c}binom{n}{a}binom{n}{b} x^c$ please
            $endgroup$
            – Salvattore
            Jul 25 '14 at 4:59












          • $begingroup$
            @Salvattore $sum_{a,b}binom{n}{a}binom{n}{b}x^{a+n-b}$, then collect all the coefficients of $x^c$ for fixed $c$ into $sum_{a+n-b=c}binom{n}{a}binom{n}{b}$ to get $sum_cleft[sum_{a+n-b=c}binom{n}{a}binom{n}{b}right]x^c$. If you want to work with generating functions, or even just plain polynomials, you need to be able to collect like terms, it is virtually a requirement.
            $endgroup$
            – anon
            Jul 29 '14 at 17:23


















          $begingroup$
          +1. @Lance C Do you see that this is the same as your "logical" proof? Choosing $n$ people from $2n$ is what you are doing when you look at the coefficient of $x^n$ in the expansion.
          $endgroup$
          – user17762
          May 23 '12 at 5:12




          $begingroup$
          +1. @Lance C Do you see that this is the same as your "logical" proof? Choosing $n$ people from $2n$ is what you are doing when you look at the coefficient of $x^n$ in the expansion.
          $endgroup$
          – user17762
          May 23 '12 at 5:12












          $begingroup$
          Can you explain me in more detail... why $sum_{a=0}^nbinom{n}{a}x^a $ * $sum_{b=0}^nbinom{n}{b}x^{n-b}$ is equal to $sum_{c=0}^{2n} sum_{a+n-b=c}binom{n}{a}binom{n}{b} x^c$ please
          $endgroup$
          – Salvattore
          Jul 25 '14 at 4:59






          $begingroup$
          Can you explain me in more detail... why $sum_{a=0}^nbinom{n}{a}x^a $ * $sum_{b=0}^nbinom{n}{b}x^{n-b}$ is equal to $sum_{c=0}^{2n} sum_{a+n-b=c}binom{n}{a}binom{n}{b} x^c$ please
          $endgroup$
          – Salvattore
          Jul 25 '14 at 4:59














          $begingroup$
          @Salvattore $sum_{a,b}binom{n}{a}binom{n}{b}x^{a+n-b}$, then collect all the coefficients of $x^c$ for fixed $c$ into $sum_{a+n-b=c}binom{n}{a}binom{n}{b}$ to get $sum_cleft[sum_{a+n-b=c}binom{n}{a}binom{n}{b}right]x^c$. If you want to work with generating functions, or even just plain polynomials, you need to be able to collect like terms, it is virtually a requirement.
          $endgroup$
          – anon
          Jul 29 '14 at 17:23






          $begingroup$
          @Salvattore $sum_{a,b}binom{n}{a}binom{n}{b}x^{a+n-b}$, then collect all the coefficients of $x^c$ for fixed $c$ into $sum_{a+n-b=c}binom{n}{a}binom{n}{b}$ to get $sum_cleft[sum_{a+n-b=c}binom{n}{a}binom{n}{b}right]x^c$. If you want to work with generating functions, or even just plain polynomials, you need to be able to collect like terms, it is virtually a requirement.
          $endgroup$
          – anon
          Jul 29 '14 at 17:23













          19












          $begingroup$

          Since $dbinom n k= dbinom n {n-k}$, the identity
          $$
          sum_{k=0}^n binom n k ^2 = binom {2n} n
          $$
          is the same as
          $$
          sum_{k=0}^n binom n k binom n {n-k} = binom {2n} n.
          $$
          So say a committee consists of $n$ Democrats and $n$ Republicans, and one will choose a subcommittee of $n$ members. One may choose $k$ Democrats and $n-k$ Republicans in $dbinom n k cdot dbinom n {n-k}$ ways. The number of Democrats is in the set ${0,1,2,ldots,n}$, thus ranging from all Republicans to all Democrats. The sum then gives the total number of ways to choose $n$ out of $2n$.






          share|cite|improve this answer









          $endgroup$


















            19












            $begingroup$

            Since $dbinom n k= dbinom n {n-k}$, the identity
            $$
            sum_{k=0}^n binom n k ^2 = binom {2n} n
            $$
            is the same as
            $$
            sum_{k=0}^n binom n k binom n {n-k} = binom {2n} n.
            $$
            So say a committee consists of $n$ Democrats and $n$ Republicans, and one will choose a subcommittee of $n$ members. One may choose $k$ Democrats and $n-k$ Republicans in $dbinom n k cdot dbinom n {n-k}$ ways. The number of Democrats is in the set ${0,1,2,ldots,n}$, thus ranging from all Republicans to all Democrats. The sum then gives the total number of ways to choose $n$ out of $2n$.






            share|cite|improve this answer









            $endgroup$
















              19












              19








              19





              $begingroup$

              Since $dbinom n k= dbinom n {n-k}$, the identity
              $$
              sum_{k=0}^n binom n k ^2 = binom {2n} n
              $$
              is the same as
              $$
              sum_{k=0}^n binom n k binom n {n-k} = binom {2n} n.
              $$
              So say a committee consists of $n$ Democrats and $n$ Republicans, and one will choose a subcommittee of $n$ members. One may choose $k$ Democrats and $n-k$ Republicans in $dbinom n k cdot dbinom n {n-k}$ ways. The number of Democrats is in the set ${0,1,2,ldots,n}$, thus ranging from all Republicans to all Democrats. The sum then gives the total number of ways to choose $n$ out of $2n$.






              share|cite|improve this answer









              $endgroup$



              Since $dbinom n k= dbinom n {n-k}$, the identity
              $$
              sum_{k=0}^n binom n k ^2 = binom {2n} n
              $$
              is the same as
              $$
              sum_{k=0}^n binom n k binom n {n-k} = binom {2n} n.
              $$
              So say a committee consists of $n$ Democrats and $n$ Republicans, and one will choose a subcommittee of $n$ members. One may choose $k$ Democrats and $n-k$ Republicans in $dbinom n k cdot dbinom n {n-k}$ ways. The number of Democrats is in the set ${0,1,2,ldots,n}$, thus ranging from all Republicans to all Democrats. The sum then gives the total number of ways to choose $n$ out of $2n$.







              share|cite|improve this answer












              share|cite|improve this answer



              share|cite|improve this answer










              answered Dec 22 '14 at 22:05









              Michael HardyMichael Hardy

              1




              1























                  12












                  $begingroup$

                  Consider the graph underlying Pascal's triangle:
                  Pascal's triangle



                  In this graph, the number at each node is a binomial coefficient and can also be thought of as the number of downward paths from the apex to that node.



                  The left side of the identity is the number of paths that start at the apex, go down to the $n$th row and return to the apex (let's call them round trips to $n$). By reflecting the return path about the $n$th row, we get a bijective correspondence between return trips to $n$ and paths from the apex to the central node of the $2n$th row. This is nothing but the central binomial coefficient $binom{2n}n$.






                  share|cite|improve this answer









                  $endgroup$


















                    12












                    $begingroup$

                    Consider the graph underlying Pascal's triangle:
                    Pascal's triangle



                    In this graph, the number at each node is a binomial coefficient and can also be thought of as the number of downward paths from the apex to that node.



                    The left side of the identity is the number of paths that start at the apex, go down to the $n$th row and return to the apex (let's call them round trips to $n$). By reflecting the return path about the $n$th row, we get a bijective correspondence between return trips to $n$ and paths from the apex to the central node of the $2n$th row. This is nothing but the central binomial coefficient $binom{2n}n$.






                    share|cite|improve this answer









                    $endgroup$
















                      12












                      12








                      12





                      $begingroup$

                      Consider the graph underlying Pascal's triangle:
                      Pascal's triangle



                      In this graph, the number at each node is a binomial coefficient and can also be thought of as the number of downward paths from the apex to that node.



                      The left side of the identity is the number of paths that start at the apex, go down to the $n$th row and return to the apex (let's call them round trips to $n$). By reflecting the return path about the $n$th row, we get a bijective correspondence between return trips to $n$ and paths from the apex to the central node of the $2n$th row. This is nothing but the central binomial coefficient $binom{2n}n$.






                      share|cite|improve this answer









                      $endgroup$



                      Consider the graph underlying Pascal's triangle:
                      Pascal's triangle



                      In this graph, the number at each node is a binomial coefficient and can also be thought of as the number of downward paths from the apex to that node.



                      The left side of the identity is the number of paths that start at the apex, go down to the $n$th row and return to the apex (let's call them round trips to $n$). By reflecting the return path about the $n$th row, we get a bijective correspondence between return trips to $n$ and paths from the apex to the central node of the $2n$th row. This is nothing but the central binomial coefficient $binom{2n}n$.







                      share|cite|improve this answer












                      share|cite|improve this answer



                      share|cite|improve this answer










                      answered Dec 16 '15 at 12:50









                      Amritanshu PrasadAmritanshu Prasad

                      1,0461013




                      1,0461013























                          11












                          $begingroup$

                          $newcommand{angles}[1]{leftlangle, #1 ,rightrangle}
                          newcommand{braces}[1]{leftlbrace, #1 ,rightrbrace}
                          newcommand{bracks}[1]{leftlbrack, #1 ,rightrbrack}
                          newcommand{ceil}[1]{,leftlceil, #1 ,rightrceil,}
                          newcommand{dd}{{rm d}}
                          newcommand{ds}[1]{displaystyle{#1}}
                          newcommand{expo}[1]{,{rm e}^{#1},}
                          newcommand{fermi}{,{rm f}}
                          newcommand{floor}[1]{,leftlfloor #1 rightrfloor,}
                          newcommand{half}{{1 over 2}}
                          newcommand{ic}{{rm i}}
                          newcommand{iff}{Longleftrightarrow}
                          newcommand{imp}{Longrightarrow}
                          newcommand{pars}[1]{left(, #1 ,right)}
                          newcommand{partiald}[3]{frac{partial^{#1} #2}{partial #3^{#1}}}
                          newcommand{pp}{{cal P}}
                          newcommand{root}[2]{,sqrt[#1]{vphantom{large A},#2,},}
                          newcommand{sech}{,{rm sech}}
                          newcommand{sgn}{,{rm sgn}}
                          newcommand{totald}[3]{frac{{rm d}^{#1} #2}{{rm d} #3^{#1}}}
                          newcommand{ul}[1]{underline{#1}}
                          newcommand{verts}[1]{leftvert, #1 ,rightvert}$

                          begin{align}
                          color{#66f}{largesum_{k = 0}^{n}{n choose k}^{2}}&=
                          sum_{k = 0}^{n}{n choose k}
                          oint_{verts{z} = 1}{pars{1 + z}^{n} over z^{k + 1}},{dd z over 2piic}
                          \[5mm] & =
                          oint_{verts{z} = 1}{pars{1 + z}^{n} over z}
                          sum_{k = 0}^{n}{n choose k}pars{1 over z}^{k},{dd z over 2piic}
                          \[5mm]&=oint_{verts{z} = 1}{pars{1 + z}^{n} over z}
                          pars{1 + {1 over z}}^{n},{dd z over 2piic}
                          \[5mm] & =
                          oint_{verts{z} = 1}{pars{1 + z}^{2n} over z^{n + 1}},{dd z over 2piic}
                          \[5mm] & =color{#66f}{large{2n choose n}}
                          end{align}






                          share|cite|improve this answer











                          $endgroup$













                          • $begingroup$
                            Can we change integral and summation? What is the reason behind that ?
                            $endgroup$
                            – Siddhant Trivedi
                            Jan 4 '17 at 4:23










                          • $begingroup$
                            @SiddhantTrivedi In this case, it's just a $finite$ sum.
                            $endgroup$
                            – Felix Marin
                            Jan 5 '17 at 22:27
















                          11












                          $begingroup$

                          $newcommand{angles}[1]{leftlangle, #1 ,rightrangle}
                          newcommand{braces}[1]{leftlbrace, #1 ,rightrbrace}
                          newcommand{bracks}[1]{leftlbrack, #1 ,rightrbrack}
                          newcommand{ceil}[1]{,leftlceil, #1 ,rightrceil,}
                          newcommand{dd}{{rm d}}
                          newcommand{ds}[1]{displaystyle{#1}}
                          newcommand{expo}[1]{,{rm e}^{#1},}
                          newcommand{fermi}{,{rm f}}
                          newcommand{floor}[1]{,leftlfloor #1 rightrfloor,}
                          newcommand{half}{{1 over 2}}
                          newcommand{ic}{{rm i}}
                          newcommand{iff}{Longleftrightarrow}
                          newcommand{imp}{Longrightarrow}
                          newcommand{pars}[1]{left(, #1 ,right)}
                          newcommand{partiald}[3]{frac{partial^{#1} #2}{partial #3^{#1}}}
                          newcommand{pp}{{cal P}}
                          newcommand{root}[2]{,sqrt[#1]{vphantom{large A},#2,},}
                          newcommand{sech}{,{rm sech}}
                          newcommand{sgn}{,{rm sgn}}
                          newcommand{totald}[3]{frac{{rm d}^{#1} #2}{{rm d} #3^{#1}}}
                          newcommand{ul}[1]{underline{#1}}
                          newcommand{verts}[1]{leftvert, #1 ,rightvert}$

                          begin{align}
                          color{#66f}{largesum_{k = 0}^{n}{n choose k}^{2}}&=
                          sum_{k = 0}^{n}{n choose k}
                          oint_{verts{z} = 1}{pars{1 + z}^{n} over z^{k + 1}},{dd z over 2piic}
                          \[5mm] & =
                          oint_{verts{z} = 1}{pars{1 + z}^{n} over z}
                          sum_{k = 0}^{n}{n choose k}pars{1 over z}^{k},{dd z over 2piic}
                          \[5mm]&=oint_{verts{z} = 1}{pars{1 + z}^{n} over z}
                          pars{1 + {1 over z}}^{n},{dd z over 2piic}
                          \[5mm] & =
                          oint_{verts{z} = 1}{pars{1 + z}^{2n} over z^{n + 1}},{dd z over 2piic}
                          \[5mm] & =color{#66f}{large{2n choose n}}
                          end{align}






                          share|cite|improve this answer











                          $endgroup$













                          • $begingroup$
                            Can we change integral and summation? What is the reason behind that ?
                            $endgroup$
                            – Siddhant Trivedi
                            Jan 4 '17 at 4:23










                          • $begingroup$
                            @SiddhantTrivedi In this case, it's just a $finite$ sum.
                            $endgroup$
                            – Felix Marin
                            Jan 5 '17 at 22:27














                          11












                          11








                          11





                          $begingroup$

                          $newcommand{angles}[1]{leftlangle, #1 ,rightrangle}
                          newcommand{braces}[1]{leftlbrace, #1 ,rightrbrace}
                          newcommand{bracks}[1]{leftlbrack, #1 ,rightrbrack}
                          newcommand{ceil}[1]{,leftlceil, #1 ,rightrceil,}
                          newcommand{dd}{{rm d}}
                          newcommand{ds}[1]{displaystyle{#1}}
                          newcommand{expo}[1]{,{rm e}^{#1},}
                          newcommand{fermi}{,{rm f}}
                          newcommand{floor}[1]{,leftlfloor #1 rightrfloor,}
                          newcommand{half}{{1 over 2}}
                          newcommand{ic}{{rm i}}
                          newcommand{iff}{Longleftrightarrow}
                          newcommand{imp}{Longrightarrow}
                          newcommand{pars}[1]{left(, #1 ,right)}
                          newcommand{partiald}[3]{frac{partial^{#1} #2}{partial #3^{#1}}}
                          newcommand{pp}{{cal P}}
                          newcommand{root}[2]{,sqrt[#1]{vphantom{large A},#2,},}
                          newcommand{sech}{,{rm sech}}
                          newcommand{sgn}{,{rm sgn}}
                          newcommand{totald}[3]{frac{{rm d}^{#1} #2}{{rm d} #3^{#1}}}
                          newcommand{ul}[1]{underline{#1}}
                          newcommand{verts}[1]{leftvert, #1 ,rightvert}$

                          begin{align}
                          color{#66f}{largesum_{k = 0}^{n}{n choose k}^{2}}&=
                          sum_{k = 0}^{n}{n choose k}
                          oint_{verts{z} = 1}{pars{1 + z}^{n} over z^{k + 1}},{dd z over 2piic}
                          \[5mm] & =
                          oint_{verts{z} = 1}{pars{1 + z}^{n} over z}
                          sum_{k = 0}^{n}{n choose k}pars{1 over z}^{k},{dd z over 2piic}
                          \[5mm]&=oint_{verts{z} = 1}{pars{1 + z}^{n} over z}
                          pars{1 + {1 over z}}^{n},{dd z over 2piic}
                          \[5mm] & =
                          oint_{verts{z} = 1}{pars{1 + z}^{2n} over z^{n + 1}},{dd z over 2piic}
                          \[5mm] & =color{#66f}{large{2n choose n}}
                          end{align}






                          share|cite|improve this answer











                          $endgroup$



                          $newcommand{angles}[1]{leftlangle, #1 ,rightrangle}
                          newcommand{braces}[1]{leftlbrace, #1 ,rightrbrace}
                          newcommand{bracks}[1]{leftlbrack, #1 ,rightrbrack}
                          newcommand{ceil}[1]{,leftlceil, #1 ,rightrceil,}
                          newcommand{dd}{{rm d}}
                          newcommand{ds}[1]{displaystyle{#1}}
                          newcommand{expo}[1]{,{rm e}^{#1},}
                          newcommand{fermi}{,{rm f}}
                          newcommand{floor}[1]{,leftlfloor #1 rightrfloor,}
                          newcommand{half}{{1 over 2}}
                          newcommand{ic}{{rm i}}
                          newcommand{iff}{Longleftrightarrow}
                          newcommand{imp}{Longrightarrow}
                          newcommand{pars}[1]{left(, #1 ,right)}
                          newcommand{partiald}[3]{frac{partial^{#1} #2}{partial #3^{#1}}}
                          newcommand{pp}{{cal P}}
                          newcommand{root}[2]{,sqrt[#1]{vphantom{large A},#2,},}
                          newcommand{sech}{,{rm sech}}
                          newcommand{sgn}{,{rm sgn}}
                          newcommand{totald}[3]{frac{{rm d}^{#1} #2}{{rm d} #3^{#1}}}
                          newcommand{ul}[1]{underline{#1}}
                          newcommand{verts}[1]{leftvert, #1 ,rightvert}$

                          begin{align}
                          color{#66f}{largesum_{k = 0}^{n}{n choose k}^{2}}&=
                          sum_{k = 0}^{n}{n choose k}
                          oint_{verts{z} = 1}{pars{1 + z}^{n} over z^{k + 1}},{dd z over 2piic}
                          \[5mm] & =
                          oint_{verts{z} = 1}{pars{1 + z}^{n} over z}
                          sum_{k = 0}^{n}{n choose k}pars{1 over z}^{k},{dd z over 2piic}
                          \[5mm]&=oint_{verts{z} = 1}{pars{1 + z}^{n} over z}
                          pars{1 + {1 over z}}^{n},{dd z over 2piic}
                          \[5mm] & =
                          oint_{verts{z} = 1}{pars{1 + z}^{2n} over z^{n + 1}},{dd z over 2piic}
                          \[5mm] & =color{#66f}{large{2n choose n}}
                          end{align}







                          share|cite|improve this answer














                          share|cite|improve this answer



                          share|cite|improve this answer








                          edited Jan 6 at 19:48

























                          answered Sep 27 '14 at 7:06









                          Felix MarinFelix Marin

                          67.5k7107141




                          67.5k7107141












                          • $begingroup$
                            Can we change integral and summation? What is the reason behind that ?
                            $endgroup$
                            – Siddhant Trivedi
                            Jan 4 '17 at 4:23










                          • $begingroup$
                            @SiddhantTrivedi In this case, it's just a $finite$ sum.
                            $endgroup$
                            – Felix Marin
                            Jan 5 '17 at 22:27


















                          • $begingroup$
                            Can we change integral and summation? What is the reason behind that ?
                            $endgroup$
                            – Siddhant Trivedi
                            Jan 4 '17 at 4:23










                          • $begingroup$
                            @SiddhantTrivedi In this case, it's just a $finite$ sum.
                            $endgroup$
                            – Felix Marin
                            Jan 5 '17 at 22:27
















                          $begingroup$
                          Can we change integral and summation? What is the reason behind that ?
                          $endgroup$
                          – Siddhant Trivedi
                          Jan 4 '17 at 4:23




                          $begingroup$
                          Can we change integral and summation? What is the reason behind that ?
                          $endgroup$
                          – Siddhant Trivedi
                          Jan 4 '17 at 4:23












                          $begingroup$
                          @SiddhantTrivedi In this case, it's just a $finite$ sum.
                          $endgroup$
                          – Felix Marin
                          Jan 5 '17 at 22:27




                          $begingroup$
                          @SiddhantTrivedi In this case, it's just a $finite$ sum.
                          $endgroup$
                          – Felix Marin
                          Jan 5 '17 at 22:27











                          1












                          $begingroup$



                          Okey this one is easy(if you seen this one once)
                          We have to show:
                          $${n choose 0}^2+{n choose 1}^2+cdots+{n choose n}^2={2n choose n} $$
                          Define a set A, $$A={a_1,ldots,a_n,a_{n+1},ldots,a_{2n}}$$ consisting of $2n$ - Elements. Now we declare $V$, is a set of $n$-sets of $A$. Obviously the cardinality of $V,$ $$|V|={2n choose n}.$$ Since one can choose $n$ Elements from $2n$ Elements in $${2n choose n}$$ ways.



                          Okey now we have the right side.
                          The for the left side you have give a disjunctive partition of the $V$ set. This should be done in the following way:
                          $V_i$ has exactly $i$-Elements from ${a_1,ldots,a_n}$, then the cardinality
                          $|V_i|={n choose i}{n choose n-i}={n choose i}{n choose i}={n choose i}^2$. Then with the Rule of Sum we have the left side. And we are done.
                          The Tricky Part ist to create the Partition to use the rule of sum.






                          share|cite|improve this answer











                          $endgroup$


















                            1












                            $begingroup$



                            Okey this one is easy(if you seen this one once)
                            We have to show:
                            $${n choose 0}^2+{n choose 1}^2+cdots+{n choose n}^2={2n choose n} $$
                            Define a set A, $$A={a_1,ldots,a_n,a_{n+1},ldots,a_{2n}}$$ consisting of $2n$ - Elements. Now we declare $V$, is a set of $n$-sets of $A$. Obviously the cardinality of $V,$ $$|V|={2n choose n}.$$ Since one can choose $n$ Elements from $2n$ Elements in $${2n choose n}$$ ways.



                            Okey now we have the right side.
                            The for the left side you have give a disjunctive partition of the $V$ set. This should be done in the following way:
                            $V_i$ has exactly $i$-Elements from ${a_1,ldots,a_n}$, then the cardinality
                            $|V_i|={n choose i}{n choose n-i}={n choose i}{n choose i}={n choose i}^2$. Then with the Rule of Sum we have the left side. And we are done.
                            The Tricky Part ist to create the Partition to use the rule of sum.






                            share|cite|improve this answer











                            $endgroup$
















                              1












                              1








                              1





                              $begingroup$



                              Okey this one is easy(if you seen this one once)
                              We have to show:
                              $${n choose 0}^2+{n choose 1}^2+cdots+{n choose n}^2={2n choose n} $$
                              Define a set A, $$A={a_1,ldots,a_n,a_{n+1},ldots,a_{2n}}$$ consisting of $2n$ - Elements. Now we declare $V$, is a set of $n$-sets of $A$. Obviously the cardinality of $V,$ $$|V|={2n choose n}.$$ Since one can choose $n$ Elements from $2n$ Elements in $${2n choose n}$$ ways.



                              Okey now we have the right side.
                              The for the left side you have give a disjunctive partition of the $V$ set. This should be done in the following way:
                              $V_i$ has exactly $i$-Elements from ${a_1,ldots,a_n}$, then the cardinality
                              $|V_i|={n choose i}{n choose n-i}={n choose i}{n choose i}={n choose i}^2$. Then with the Rule of Sum we have the left side. And we are done.
                              The Tricky Part ist to create the Partition to use the rule of sum.






                              share|cite|improve this answer











                              $endgroup$





                              Okey this one is easy(if you seen this one once)
                              We have to show:
                              $${n choose 0}^2+{n choose 1}^2+cdots+{n choose n}^2={2n choose n} $$
                              Define a set A, $$A={a_1,ldots,a_n,a_{n+1},ldots,a_{2n}}$$ consisting of $2n$ - Elements. Now we declare $V$, is a set of $n$-sets of $A$. Obviously the cardinality of $V,$ $$|V|={2n choose n}.$$ Since one can choose $n$ Elements from $2n$ Elements in $${2n choose n}$$ ways.



                              Okey now we have the right side.
                              The for the left side you have give a disjunctive partition of the $V$ set. This should be done in the following way:
                              $V_i$ has exactly $i$-Elements from ${a_1,ldots,a_n}$, then the cardinality
                              $|V_i|={n choose i}{n choose n-i}={n choose i}{n choose i}={n choose i}^2$. Then with the Rule of Sum we have the left side. And we are done.
                              The Tricky Part ist to create the Partition to use the rule of sum.







                              share|cite|improve this answer














                              share|cite|improve this answer



                              share|cite|improve this answer








                              edited Jul 10 '17 at 21:06









                              Michael Hardy

                              1




                              1










                              answered Jul 3 '17 at 17:27









                              thethathetha

                              268115




                              268115























                                  0












                                  $begingroup$

                                  Imagine that we are distributing $n$ indistinguishable candies to $k$ distinguishable children. By a direct application of Balls and Urns, there are $${n+k-1choose k-1}$$ ways to do this. Alternatively, we can first give $0le ile n$ candies to the oldest child so that we are essentially giving $n-i$ candies to $k-1$ kids and again, with Balls and Urns, $${n+k-1choose k-1}=sum_{i=0}^n{n+k-2-ichoose k-2}$$, which simplifies to the desired result.






                                  share|cite|improve this answer









                                  $endgroup$


















                                    0












                                    $begingroup$

                                    Imagine that we are distributing $n$ indistinguishable candies to $k$ distinguishable children. By a direct application of Balls and Urns, there are $${n+k-1choose k-1}$$ ways to do this. Alternatively, we can first give $0le ile n$ candies to the oldest child so that we are essentially giving $n-i$ candies to $k-1$ kids and again, with Balls and Urns, $${n+k-1choose k-1}=sum_{i=0}^n{n+k-2-ichoose k-2}$$, which simplifies to the desired result.






                                    share|cite|improve this answer









                                    $endgroup$
















                                      0












                                      0








                                      0





                                      $begingroup$

                                      Imagine that we are distributing $n$ indistinguishable candies to $k$ distinguishable children. By a direct application of Balls and Urns, there are $${n+k-1choose k-1}$$ ways to do this. Alternatively, we can first give $0le ile n$ candies to the oldest child so that we are essentially giving $n-i$ candies to $k-1$ kids and again, with Balls and Urns, $${n+k-1choose k-1}=sum_{i=0}^n{n+k-2-ichoose k-2}$$, which simplifies to the desired result.






                                      share|cite|improve this answer









                                      $endgroup$



                                      Imagine that we are distributing $n$ indistinguishable candies to $k$ distinguishable children. By a direct application of Balls and Urns, there are $${n+k-1choose k-1}$$ ways to do this. Alternatively, we can first give $0le ile n$ candies to the oldest child so that we are essentially giving $n-i$ candies to $k-1$ kids and again, with Balls and Urns, $${n+k-1choose k-1}=sum_{i=0}^n{n+k-2-ichoose k-2}$$, which simplifies to the desired result.







                                      share|cite|improve this answer












                                      share|cite|improve this answer



                                      share|cite|improve this answer










                                      answered Jan 16 '18 at 3:44









                                      DigammaDigamma

                                      6,1621440




                                      6,1621440






























                                          draft saved

                                          draft discarded




















































                                          Thanks for contributing an answer to Mathematics Stack Exchange!


                                          • Please be sure to answer the question. Provide details and share your research!

                                          But avoid



                                          • Asking for help, clarification, or responding to other answers.

                                          • Making statements based on opinion; back them up with references or personal experience.


                                          Use MathJax to format equations. MathJax reference.


                                          To learn more, see our tips on writing great answers.




                                          draft saved


                                          draft discarded














                                          StackExchange.ready(
                                          function () {
                                          StackExchange.openid.initPostLogin('.new-post-login', 'https%3a%2f%2fmath.stackexchange.com%2fquestions%2f148583%2fcombinatorial-proof-of-summation-of-sum-limits-k-0n-n-choose-k2-2n%23new-answer', 'question_page');
                                          }
                                          );

                                          Post as a guest















                                          Required, but never shown





















































                                          Required, but never shown














                                          Required, but never shown












                                          Required, but never shown







                                          Required, but never shown

































                                          Required, but never shown














                                          Required, but never shown












                                          Required, but never shown







                                          Required, but never shown







                                          Popular posts from this blog

                                          Plaza Victoria

                                          In PowerPoint, is there a keyboard shortcut for bulleted / numbered list?

                                          How to put 3 figures in Latex with 2 figures side by side and 1 below these side by side images but in...